1answer.
Ask question
Login Signup
Ask question
All categories
  • English
  • Mathematics
  • Social Studies
  • Business
  • History
  • Health
  • Geography
  • Biology
  • Physics
  • Chemistry
  • Computers and Technology
  • Arts
  • World Languages
  • Spanish
  • French
  • German
  • Advanced Placement (AP)
  • SAT
  • Medicine
  • Law
  • Engineering
castortr0y [4]
3 years ago
9

A coach ordered 1,565 flying discs for field day. The order is shipped in 5 boxes, with the same number of flying discs in each

box. How many flying discs are in each box?
Mathematics
1 answer:
Kruka [31]3 years ago
5 0

Answer: 313 in each box.

Step-by-step explanation: So you have 1,565 flying discs, and they are DIVIDED into 5 boxes, you would divide 1,565 by 5, with that, you keep on subtracting down to get to where you cannot subtract anymore, which gets you down to your answer!

NOTE: I was in class at this time, so that's why I didn't type as much as I should! Your welcome!  

You might be interested in
(4xy^2+6y)dx+(5yx^2+8x)dy=0<br> solve by using integrating factor
RSB [31]
Are u sure about the second arrow?BTW
5 0
4 years ago
20. Carmen can buy bottles of paint for $2.00 each and boxes of colored pencils for $3.50 each. She can spend no more than
EleoNora [17]

Answer:

(a) The equality that express many bottles of paint, x, and boxes of colored pencils, y, Carmen can buy is  2 x + 3.5 y = 42

b)Three different solutions:

x  = 14, y = 4 is First Solution.

x  = 7, y = 8 is Second Solution.

x  = 21, y = 0 is Third Solution.

Step-by-step explanation:

Here, the cost of 1 bottle of paint = $2.00 each

The cost of 1 box of colored pencils  = $3.5 each

Let us assume the number of bottle of paints purchased = x

So, the cost of x bottle of paints   = x ( Cost of 1 bottle of paint)

= x ($2.00)  = 2 x

Also, assume the number of box of colored pencils purchased = y

So, the cost of y box of colored pencils = y ( Cost of 1 box)

= y ($3.50)  = 3.5 y

Also, the total amount to be spent on art supplies  = $42

So, the total amount spent on x paint bottles + y box of colored pencil

= $42

or,  2 x + 3.5 y = 42

a ) So, the equality that express many bottles of paint, x, and boxes of colored pencils, y, Carmen can buy is  2 x + 3.5 y = 42

b)Three different solutions:

When y = 4 ,  equation is:   2 x + 3.5(4)  = 42

or,  2 x = 42 - 14  = 28, or x = 28/2  =  14

So, x  = 14, y = 4 is First Solution.

When y = 8 ,  equation is:   2 x + 3.5(8)  = 42

or,  2 x = 42 - 28  = 14, or x = 14/2  =  7

So, x  = 7, y = 8 is Second Solution.

When y = 0 ,  equation is:   2 x + 3.5(0)  = 42

or,  2 x = 42 , or x = 42/2  = 21

So, x  = 21, y = 0 is Third Solution.

7 0
3 years ago
(7 x 18 + 45) : 3 x 2 what is it please?​
11Alexandr11 [23.1K]

The Answer Is 171 : 6

Hope This Help's

7 0
3 years ago
Not sure what to put in they yellow boxes?
Elza [17]

Answer:

Hamburger=31%

Pie=47%

Hot Dog=22%

36=100%

Step-by-step explanation:

What you do is you take the number of how many people have taken the survey and put it over 36.

11/36, 17/36, 8/36=

Hamburger=31%

Pie=47%

Hot Dog=22%

36=100%

8 0
3 years ago
Determine the remainder when (5x3 + 49x2 + 74x + 6) is divided by (x + 8).
blondinia [14]
The polynomial remainder theorem says that the remainder when dividing a polynomial P(x) by a linear divisor x-k is simply P(k).

If P(x)=5x^3+49x^2+74x+6, then the remainder upon dividing by x+8 is

P(-8)=-10

You could also verify this by actually computing the quotient and remainder.
4 0
3 years ago
Read 2 more answers
Other questions:
  • Help Geometry I need this answer please
    11·1 answer
  • Which expression is equal to 3(56)? 3(50 + 6) 3(50) + (6) 5(30 + 6) 5(30) + 5(6)
    11·2 answers
  • Tara's science project was graded based on creativity, accuracy, and effort. If she received a 9.4 for each of the three citeria
    9·1 answer
  • What is the tangential acceleration (i.e., the acceleration parallel to the road) of car b?
    10·1 answer
  • The coordinate grid shows three points P, Q, and R:
    7·1 answer
  • Z − 4 = 8<br> Solve for z.
    6·2 answers
  • Guys please help me!!
    10·1 answer
  • WILL MARK BRAINLIEST. PLEASE ANSWER FAST
    12·1 answer
  • Write 750% as a decimal and as a fraction in simplest form
    12·1 answer
  • You borrowed $10,400 for 4 years at 12.7% and the interest is compounded annually. What is the total you will pay back?
    7·1 answer
Add answer
Login
Not registered? Fast signup
Signup
Login Signup
Ask question!